Fiche de mathématiques
> >

Calcul matriciel : Exercices

Partager :


Dans l'ensemble de cette fiche, n est un entier naturel non nul sauf mention contraire.

exercice 1

Dire si les matrices suivantes sont inversibles et donner leurs inverses.

1. \begin{pmatrix} 1 & 1 & -1\\2 & 0 & 1\\ 2 & 1 & -1\end{pmatrix}

2. \begin{pmatrix}1 & 0 & 1\\2 & -1 & 1\\-1 & 1 & -1 \end{pmatrix}




exercice 2

Résoudre dans \mathcal{M}_2(\mathbb{R}) l'équation matricielle suivante :

X^2+X=\begin{pmatrix}1 & 1\\1 & 1\end{pmatrix}




exercice 3

Soit A=\begin{pmatrix}-1 & -2\\3 & 4\end{pmatrix}

1. Calculer A^2-3A+2I. Montrer que A est inversible et calculer son inverse

2. Soit n\in \mathbb{N}^{*}-\lbrace 1\rbrace
Déterminer le reste de la division euclidienne de X^n par X^2-3X+2 et en déduire A^n.




exercice 4

Déterminer une CNS pour que le produit de deux matrices symétriques soit une matrice symétrique.




exercice 5

Montrer que \mathcal{S}_n(\mathbb{R})\oplus\mathcal{A}_n(\mathbb{R})=\mathcal{M}_n(\mathbb{R}), avec \mathcal{S}_n(\mathbb{R}) le sous espace des matrices symétriques et \mathcal{A}_n(\mathbb{R}) celui des matrices antisymétriques.




exercice 6

Soit n\in\mathbb{N}^*. Pour \sigma\in S_n On pose P_{\sigma}=\left(\delta_{i,\sigma(j)}\right)_{1\le i,j\le n}, avec \rm\delta_{i,j}=\left \lbrace \begin{array}{l}1\ {\rm si}\ i=j\\0\ {\rm si}\  i\neq j\end{array} \right. appelé symbole de Kronecker.

1. Dans le cas particulier de n=4, calculer P_{\sigma} pour \sigma=(1\quad 2) puis pour \sigma=(1\quad 2)(3\quad 4) .

2. Pour tout \sigma,\sigma'\in S_n, comparer P_{\sigma}P_{\sigma'} et P_{\sigma o\sigma'}.

3. Calculer P_{Id} puis montrer que E=\lbrace P_{\sigma}/ \sigma\in S_n\rbrace est un sous-groupe de \mathcal{G}L_n(\mathbb{R}).

4. E et S_n sont-ils isomorphes?

5.Montrer que: \forall \sigma\in S_n\text{ : } ^tP_{\sigma}=P_{\sigma^{-1}}


exercice 7

Soit A\in\mathcal{M}_n(\mathbb{K}) telle que A=diag(\lambda_1,\cdots,\lambda_n) avec \lambda_1,\ldots,\lambda_n\in\mathbb{K} tels que \forall i,j\in \ldbrack1,n\rdbrack : \lambda_i \neq \lambda_j et soit:
\begin{array}{cccc} f:&\mathcal{M}_n(\mathbb{K})& \to &\mathcal{M}_n(\mathbb{K})\\   & M & \to & MA-AM  \\ \end{array}

Montrer que Im(f) est l'ensemble des matrices à diagonale nulle.




exercice 8

Soit A=(a_{ij})_{1\le i,j\le n+1} une matrice de \mathcal{M}_{n+1}(\mathbb{R})a_{i,j}={j-1\choose i-1} pour tout i,j de \ldbrack1,n\rdbrack et soit f\in\mathcal{L}(\mathbb{R}_n[X]) exprimé par la matrice A dans la base canonique de \mathbb{R}_n[X]

1.Exprimer f(P) pour tout polynôme P de degré inférieur ou égal à n

2. Calculer  A^m \text{ pour } m\in\mathbb{N} puis A^{-1}




exercice 9

Calculer le rang en fonction des réels a,b,c et d des matrices suivantes:

1. A=\begin{pmatrix}1 & \dfrac{1}{2} & \dfrac{1}{3} \\\dfrac{1}{2} & \dfrac{1}{3} & \dfrac{1}{4} \\ \dfrac{1}{3} & \dfrac{1}{4} & a \end{pmatrix}
2. B=\begin{pmatrix}1 & a & 1 & b\\a & 1 & b & 1\\ 1 & b & 1 & a\\b & 1 & a & 1\end{pmatrix}
3. C=\rm\begin{pmatrix}1 & 1 & 1 \\b+c & c+a & a+b \\ bc & ca & ab \end{pmatrix}




exercice 10

Soit \rm M\in\mathcal{A}_3(\mathbb{K}). Quel est le rang de M ?




exercice 11

Soient (n,p,r)\in(\mathbb{N}^*)^3 , A\in\mathcal{M}_{n,p}(\mathbb{K}) de rang r.
Montrer qu'il existe B\in\mathcal{M}_{n,r}(\mathbb{K}) et C\in\mathcal{M}_{r,p}(\mathbb{K}) telles que A=BC.




exercice 12

Montrer que toute matrice triangulaire supérieure de \mathcal{M}_n(\mathbb{K}) est semblable à une matrice triangulaire inférieure.



exercice 1

1. La matrice est inversible car son déterminant est 1 non nul.
Calcul de l'inverse:

\left(\begin{array}{ccc|ccc} 1&1&-1 &1&0&0\\ 2&0&1 &0&1&0\\ 2&1&-1 &0&0&1\\ \end{array}\right)\underbrace{\rightarrow}_{L_2\leftrightarrow 2L_1-L_2}\left(\begin{array}{ccc|ccc} 1&1&-1 &1&0&0\\ 0&2&-3 &2&-1&0\\ 2&1&-1 &0&0&1\\ \end{array}\right)\underbrace{\rightarrow}_{L_3\leftrightarrow 2L_1-L_3}\left(\begin{array}{ccc|ccc} 1&1&-1 &1&0&0\\ 0&2&-3 &2&-1&0\\ 0&1&-1 &2&0&-1\\ \end{array}\right)
\underbrace{\rightarrow}_{L_3\leftrightarrow L_2-2L_3}\left(\begin{array}{ccc|ccc} 1&1&-1 &1&0&0\\ 0&2&-3 &2&-1&0\\ 0&0&-1 &-2&-1&2\\ \end{array}\right)\underbrace{\rightarrow}_{L_3\leftrightarrow -L_3}\left(\begin{array}{ccc|ccc} 1&1&-1 &1&0&0\\ 0&2&-3 &2&-1&0\\ 0&0&1 &2&1&-2\\ \end{array}\right)\underbrace{\rightarrow}_{L_2\leftrightarrow L_2+3L_3}&\left(\begin{array}{ccc|ccc} 1&1&-1 &1&0&0\\ 0&2&0 &8&2&-6\\ 0&0&1 &2&1&-2\\ \end{array}\right)
\underbrace{\rightarrow}_{L_1\leftrightarrow L_1+L_3}&\left(\begin{array}{ccc|ccc} 1&1& 0&3&1&-2\\ 0&2&0&8&2&-6\\ 0&0&1 &2&1&-2\\ \end{array}\right)\underbrace{\rightarrow}_{L_2\leftrightarrow \frac{1}{2}L_2}&\left(\begin{array}{ccc|ccc} 1&1& 0&3&1&-2\\ 0&1&0&4&1&-3\\ 0&0&1 &2&1&-2\\ \end{array}\right)\underbrace{\rightarrow}_{L_1\leftrightarrow L_1-L_2}&\left(\begin{array}{ccc|ccc} 1&0& 0&-1&0&1\\ 0&1&0&4&1&-3\\ 0&0&1 &2&1&-2\\ \end{array}\right)\end{matrix}

Resultat:
\boxed{\text{L'inverse de la matrice est donc : } \begin{pmatrix}-1&0&1\\4&1&-3\\2&1&-2\end{pmatrix}}

2. La matrice est inversible car son déterminant est 1 non nul.
Calcul de l'inverse:

\left(\begin{array}{ccc|ccc} 1&0&1 &1&0&0\\ 2&-1&1 &0&1&0\\ -1&1&-1 &0&0&1\\ \end{array}\right)\underbrace{\rightarrow}_{L_2\leftrightarrow 2L_1-L_2}\left(\begin{array}{ccc|ccc} 1&0&1 &1&0&0\\ 0&1&1 &2&-1&0\\ -1&1&-1 &0&0&1\\ \end{array}\right)\underbrace{\rightarrow}_{L_3\leftrightarrow L_1+L_3}\left(\begin{array}{ccc|ccc} 1&0&1 &1&0&0\\ 0&1&1 &2&-1&0\\ 0&1&0 &1&0&1\\ \end{array}\right)
\underbrace{\rightarrow}_{L_3\leftrightarrow L_2-L_3}\left(\begin{array}{ccc|ccc} 1&0&1 &1&0&0\\ 0&1&1 &2&-1&0\\ 0&0&1 &1&-1&-1\\ \end{array}\right)\underbrace{\rightarrow}_{L_2\leftrightarrow L_2-L_3}\left(\begin{array}{ccc|ccc} 1&0&1 &1&0&0\\ 0&1&0 &1&0&1\\ 0&0&1 &1&-1&-1\\ \end{array}\right)\underbrace{\rightarrow}_{L_1\leftrightarrow L_1-L_3}\left(\begin{array}{ccc|ccc} 1&0&0 &0&1&1\\ 0&1&0 &1&0&1\\ 0&0&1 &1&-1&-1\\ \end{array}\right)

Resultat:
\boxed{\text{L'inverse de la matrice est donc : } \begin{pmatrix}0&1&1\\1&0&1\\1&-1&-1\end{pmatrix}}


exercice 2

On pose  X=\begin{pmatrix}a&b\\c&d\end{pmatrix}, donc :
X^2+X=\begin{pmatrix}1&1\\1&1\end{pmatrix} \Longleftrightarrow  \left \lbrace \begin{array}{c @{ = } c} a^2+bc+a&1 \\  ab+bd+b&1 \\ca+dc+c&1\\  cb+d^2+d&1   \\\end{array} \right. \Longleftrightarrow  \left \lbrace \begin{array}{c @{ = } c} a^2+bc+a&1 \\  b(a+d+1)&1 \\c(a+d+1)&1\\  (d-a)(a+d+1)&0   \\\end{array} \right.\Longleftrightarrow  \left \lbrace \begin{array}{c @{ = } c} d&a\neq -\dfrac{1}{2} \\  c&b= \dfrac{1}{2a+1} \\a^2+\dfrac{1}{(2a+1)^2}+a&1\\ \end{array} \right.
D'autre part:
 a^2+\dfrac{1}{(2a+1)^2}+a=1 \Longleftrightarrow 4a^4+8a^3+a^2-3a=0\Longleftrightarrow a\in\left\lbrace -\dfrac{3}{2},-1,0,\dfrac{1}{2}\right \rbrace

Donc, l'ensemble des solutions est:
 \boxed{S= \left\lbrace \begin{pmatrix}-1&-1\\-1&-1\end{pmatrix}\text{ , } \begin{pmatrix}0&1\\1&0\end{pmatrix}\text{ , }\begin{pmatrix}\dfrac{1}{2}&\dfrac{1}{2} \\ \dfrac{1}{2}&\dfrac{1}{2}\end{pmatrix}\text{ , }\begin{pmatrix}-\dfrac{3}{2}&-\dfrac{1}{2} \\ -\dfrac{1}{2}& -\dfrac{3}{2}\end{pmatrix} \right\rbrace }


exercice 3

1. A^2-3A+2I=\begin{pmatrix}-5&-6\\9&10\end{pmatrix}-3\begin{pmatrix}-1&-2\\3&4\end{pmatrix}+2\begin{pmatrix}1&0\\0&1\end{pmatrix}=\begin{pmatrix}0&0\\0&0\end{pmatrix}
Donc:
\boxed{A^2-3A+2I=0}

Ainsi:
3A-A^2=2I \Longleftrightarrow A(3I-A)=2I \Longleftrightarrow A \left(\dfrac{3}{2}I - \dfrac{1}{2}A \right)=I

D'où:
\boxed{\text{ A est inversible avec } A^{-1} = \dfrac{3}{2}I-\dfrac{1}{2}A}

2. En notant Q le polynome quotient, il existe a, b réels tels que: X^n=(X^2-3X+2)Q(X)+aX+b
Les deux racines de X^2-3X+2=0 sont 1 et 2, donc en évaluant successivement ces deux racines, on obtient:
\left \lbrace \begin{array}{c @{ = } c} 1 & a+b \\ 2^n & 2a+b \\ \end{array} \right

La résolution de ce système donne alors [a=-1+2^n,\quad b=2-2^n]
\boxed{ \text{Le reste de la division euclidienne est donc: } (2^n-1)X+2-2^n }

Or, on sait que A^2-3A+2I=0 d'où:
\boxed{A^n=(-1+2^n)A+(2-2^n)I}


exercice 4

Soit (A,B)\in \mathcal{S}_n(\mathbb{R})^2
On a:
\text{ AB est symétrique }\Longleftrightarrow AB=^t(AB)\Longleftrightarrow AB=^tB^tA\Longleftrightarrow AB=BA\Longleftrightarrow \text{A et B commutent}

CNS:
\boxed{\text{Le produit de deux matrices symétriques est une matrice symétrique si, et seulement si, les deux matrices commutent}}


exercice 5

Soit M\in \mathcal{S}_n(\mathbb{R})\cap A_n(\mathbb{R})
Alors:
\left \lbrace \begin{array}{c @{ = } c} ^tM & M \\ ^tM & -M \\ \end{array} \right.

Ainsi M=-M et donc M = 0
On en déduit que:
\boxed{\mathcal{S}_n(\mathbb{R})\cap \mathcal{A}_n(\mathbb{R})=\lbrace 0\rbrace } ~(I)

Soit M\in\mathcal{M}_n(\mathbb{R})
La matrice M peut s'écrire:
M = \dfrac{M+^tM}{2}+\dfrac{M-^tM}{2}

Notons:
M_1=\dfrac{M+^tM}{2} \text{ et } M_2=\dfrac{M-^tM}{2}

On a:
\begin{cases}^tM_1=^t\left(\dfrac{M+^tM}{2}\right) = \dfrac{1}{2}(^t(M+^tM)) =\dfrac{M+^tM}{2}=M_1\\ ^tM_2=^t\left(\dfrac{M-^tM}{2}\right) = \dfrac{1}{2}(^t(M-^tM)) =\dfrac{^tM-M}{2}=-\dfrac{M-^tM}{2}=-M_2\end{cases}

Donc:
M_1\in \mathcal{S}_n(\mathbb{R}) \text{ et } M_2\in \mathcal{A}_n(\mathbb{R})

On en déduit que:
\boxed{\text{ Pour toute matrice M de } \mathcal{M}_n(\mathbb{R}), \text{ il existe } M_1\in \mathcal{S}_n(\mathbb{R}) \text{ et } M_2\in \mathcal{A}_n(\mathbb{R}) \text{ telles que: } M=M_1+M_2}~(II)

De (I) et (II):
\boxed{\mathcal{S}_n(\mathbb{R}) \oplus \mathcal{A}_n(\mathbb{R})=\mathcal{M}_n(\mathbb{R})}
.

exercice 6

1.
Pour \sigma =(1\quad 2)
On écrit matriciellement:
\sigma=\begin{pmatrix} 1&2&3&4\\2&1&3&4\end{pmatrix}

On a donc:
\sigma(1)=2\text{ , } \sigma(2)=1\text{ , } \sigma(3)=3 \text{ et } \sigma(4)=4

Ceci nous permet de calculer:
P_{\sigma}=\left(\delta_{i,\sigma(j)}\right)_{1\le i,j\le 4}=\begin{pmatrix} \delta_{1,\sigma(1)&\delta_{1,\sigma(2)&\delta_{1,\sigma(3)&\delta_{1,\sigma(4)\\\delta_{2,\sigma(1)&\delta_{2,\sigma(2)&\delta_{2,\sigma(3)&\delta_{2,\sigma(4)\\\delta_{3,\sigma(1)&\delta_{3,\sigma(2)&\delta_{3,\sigma(3)&\delta_{3,\sigma(4)\\\delta_{4,\sigma(1)&\delta_{4,\sigma(2)&\delta_{4,\sigma(3)&\delta_{4,\sigma(4)\end{pmatrix}=\begin{pmatrix} \delta_{1,2&\delta_{1,1&\delta_{1,3&\delta_{1,4\\\delta_{2,2&\delta_{2,1&\delta_{2,3&\delta_{2,4\\\delta_{3,2&\delta_{3,1&\delta_{3,3&\delta_{3,4\\\delta_{4,2&\delta_{4,1&\delta_{4,3&\delta_{4,4\end{pmatrix}

On en déduit:
\boxed{P_{\sigma}=\begin{pmatrix} 0&1&0&0\\1&0&0&0\\0&0&1&0\\0&0&0&1\end{pmatrix}}

Pour \sigma=(1\quad 2)(3\quad 4)
De la même manière, on écrit matriciellement:
\sigma=\begin{pmatrix} 1&2&3&4\\2&1&4&3\end{pmatrix}

On a donc:
\sigma(1)=2\text{ , } \sigma(2)=1\text{ , } \sigma(3)=4 \text{ et } \sigma(4)=3

On trouve:
\boxed{P_{\sigma}=\begin{pmatrix} 0&1&0&0\\1&0&0&0\\0&0&0&1\\0&0&1&0\end{pmatrix}}

2. Notons \mathcal{B} = (e_1,\ldots,e_n) la base canonique de \mathbb{R}_n et notons f l'endomorphisme canoniquement associé à P_{\sigma}.
On a donc:
\forall j \in \ldbrack 1,n\rdbrack \text{ : } f_{\sigma}(e_j) = e_{\sigma(j)}

Soient \sigma,\sigma'\in S_n, on a:
\forall j \in \ldbrack 1,n\rdbrack \text{ : }(f_{\sigma}\circ  f_{\sigma'} )(e_j) =f_{\sigma}\left(f_{\sigma'} (e_j)\right)=f_{\sigma}(e_{\sigma'(j)})=e_{\sigma(\sigma'(j))}=e_{\sigma\circ\sigma'(j)}=f_{\sigma\circ\sigma'}(e_j)

On conclut alors que:
\boxed{P_{\sigma\circ\sigma'}=P_{\sigma}P_{\sigma'}}

3. On a:
P_{Id}=(\delta_{i,Id(j)})_{1\le i,j\le n}=(\delta_{i,j})_{1\le i,j\le n}

Donc:
\boxed{P_{Id}=I_n}

L'ensemble E est non vide puisqu'il contient la matrice identité.
On a montré dans la question 2. que E est stable par composition.
De plus:
P_{\sigma}P_{\sigma^{-1}}=P_{\sigma o\sigma^{-1}}= P_{Id}=I_n

Ce qui veut dire que:
P_{\sigma}\in\mathcal{G}L_n(\mathbb{R})\text{ et } P_{\sigma}^{-1}=P_{\sigma^{-1}}\in E

Ainsi:
\boxed{\text{E est un sous-groupe de } \mathcal{G}L_n(\mathbb{R})}

4. On considère l'application:
P : S_n\to E\\~~~~~~~ \sigma \mapsto P_\sigma

Cette application étant clairement un morphisme de groupe surjective, montrons qu'elle est injective:
\sigma \in Ker(P) \Longleftrightarrow P_\sigma = I_n \Longleftrightarrow \forall j\in \ldbrack1,n\rdbrack\text{ : } \sigma(j)=j \Longleftrightarrow \sigma=Id

P est donc un isomorphisme, et:
\boxed{\text{ E est isomorphe à } S_n}

5. Soit \sigma\in S_n, on a donc:
^tP_{\sigma}=(\delta_{j,\sigma(i)})_{1\le i,j\le n}=(\delta_{\sigma^{-1}(j),i})_{1\le i,j\le n}=(\delta_{i,\sigma^{-1}(j)})_{1\le i,j\le n}=P_{\sigma^{-1}}

Conclusion:
\boxed{\forall \sigma\in S_n\text{ : } ^tP_{\sigma}=P_{\sigma^{-1}}}


exercice 7


Soit N\in \mathcal{M}_n(\mathbb{K})
On a:
N \in Im(f) \Longleftrightarrow \exists M \in \mathcal{M}_n(\mathbb{K}) / N = f(M) \Longleftrightarrow \exists M \in \mathcal{M}_n(\mathbb{K}) / N = MA-AM

Posons M=(m_{ij})_{1\le i,j\le n}, on a donc:
\begin{matrix}N&=&MA - AM\\&=&(m_{ij})_{1\le i,j\le n} diag(\lambda_1,\cdots,\lambda_n)-diag(\lambda_1,\cdots,\lambda_n)(m_{ij})_{1\le i,j\le n}\\&=&(\lambda_j m_{ij})_{1\le i,j\le n}-(\lambda_i m_{ij})_{1\le i,j\le n}\\&=&\left((\lambda_i-\lambda_j)m_{ij}\right)_{1\le i,j\le n}\end{matrix}

Donc:
 \text{Si } N\in Im(f) \text{ alors N est de diagonale nulle}

Ainsi:
\boxed{Im(f)\subset  \left\lbrace (m_{ij})_{1\le i,j\le n}~/~ \forall i\in \ldbrack1,n\rdbrack \text{ : } m_{ii}=0 \right\rbrace }~~(I)

Soit M=(m_{ij})_{1\le i,j\le n}\in \mathcal{M}_n(\mathbb{K})
On a:
M\in Ker(f) \Longleftrightarrow AM = MA \Longleftrightarrow \forall i,j\in \ldbrack1,n\rdbrack \text{ : }\lambda_im_{ij}=\lambda_jm_{ij}\Longleftrightarrow m_{ij}=0 \text{ pour tout } i\neq j \Longleftrightarrow M\in \mathcal{D}_n(\mathbb{K})

Donc:
Ker(f)=\mathcal{D}_n(\mathbb{K}) \text{ et } \dim\left(Ker(f)\right)=n

Donc, d'après le théorème du rang:
\dim(Im(f))=n^2-n

Il s'ensuit que:
\boxed{\dim(Im(f))=\dim\left(\left\lbrace (m_{ij})_{1\le i,j\le n}~/~ \forall i\in \ldbrack1,n\rdbrack \text{ : } m_{ii}=0 \right\rbrace }\right)}~~(II)

De (I) et (II):
\boxed{ Im(f) \text{ est l'ensemble des matrices à diagonale nulle }}


exercice 8

1. Pour k appartenant à \lbrace 0,\ldots,n\rbrace, on a:
f(X^k) = \displaystyle \sum_{i=0}^n{k\choose i}X^i = \displaystyle \sum_{i=0}^k{k\choose i}X^i=(X+1)^k

Donc par linéarité:
\boxed{\forall P\in \mathbb{K}_n[X]~:~f(P)=P(X+1)}

2. Soit m un entier naturel, on a:
f^m(X^k)=(X+m)^k = \displaystyle \sum_{i=0}^n{k\choose i}m^{k-i}X^i

Donc:
\boxed{A^m=(m^{j-i}a_{ij})_{1\le i,j\le n+1}}

De même on montre que:
\boxed{A^{-1}=((-1)^{j-i}a_{ij})_{1\le i,j\le n+1}}


exercice 9

1. rg(A)= rg\begin{pmatrix}1 & \dfrac{1}{2} & \dfrac{1}{3} \\\dfrac{1}{2} & \dfrac{1}{3} & \dfrac{1}{4} \\ \dfrac{1}{3} & \dfrac{1}{4} & a \end{pmatrix} = rg\begin{pmatrix}1 & 0 & 0 \\\dfrac{1}{2} & \dfrac{1}{12} & \dfrac{1}{12} \\ \dfrac{1}{3} & \dfrac{1}{12} & a-\dfrac{1}{9} \end{pmatrix}= rg\begin{pmatrix}1 & 0 & 0 \\\dfrac{1}{2} & \dfrac{1}{12} & 0 \\ \dfrac{1}{3} & \dfrac{1}{12} & a-\dfrac{7}{36} \end{pmatrix}

\left(\text{ Opérations: } (C1,C2,C3) \rightarrow (C1,C2-\frac{1}{2}C1,C3-\frac{1}{3}C1) \rightarrow (C1,C2,C3-C2)~\right)

Si a =\dfrac{7}{36} , alors rg(A)=2
Si a \neq\dfrac{7}{36} , alors rg(A)=3
\boxed{ rg(A)=\begin{cases} 2\text{ si } a =\dfrac{7}{36} \\ 3\text{ si } a \neq\dfrac{7}{36} \end{cases}}

2. rg(B)=rg\begin{pmatrix}1 & a & 1 & b\\a & 1 & b & 1\\ 1 & b & 1 & a\\b & 1 & a & 1\end{pmatrix}=rg\begin{pmatrix}1 & 0 & 0 & 0\\a & 1-a^2 & b-a & 1-ab\\ 1 & b-a & 0 & a-b\\b & 1-ab & a-b & 1-b^2\end{pmatrix}=1+rg\begin{pmatrix} 1-a^2 & b-a & 1-ab\\  b-a & 0 & a-b\\ 1-ab & a-b & 1-b^2\end{pmatrix}

\left(\text{ Opérations: } (C1,C2,C3,C4) \rightarrow (C1,C2-aC1,C3-C1,C4-bC1)~\right)

Deux cas se présentent:

\bullet\text{ Premiere cas: } a\neq b

rg(B)=1+rg\begin{pmatrix} 1-a^2 & b-a & 1-ab\\  b-a & 0 & a-b\\ 1-ab & a-b & 1-b^2\end{pmatrix}=1+rg\begin{pmatrix} 1-a^2 & 1 & 1-ab\\  1 & 0 & -1\\ 1-ab & -1 & 1-b^2\end{pmatrix}=1+rg\begin{pmatrix}  1 & 0 & -1\\ 1-a^2 & 1 & 1-ab\\ 1-ab & -1 & 1-b^2\end{pmatrix}=1+rg\begin{pmatrix}  1 & 0 & 0\\ 1-a^2 & 1 & 2-a^2-ab\\ 1-ab & -1 & 2-b^2-ab\end{pmatrix}

\left(\text{ Opérations: } (L1,L2,L3) \rightarrow (L1,\frac{1}{b-a}L2,L3) \rightarrow (L2,L1,L3) \rightarrow (C1,C2,C1+C3)~\right)
Ce qui donne:
rg(B)=2+rg\begin{pmatrix}  1 & 2-a^2-ab\\-1 & 2-b^2-ab\end{pmatrix}=2+rg\begin{pmatrix}  1 & 2-a^2-ab\\0 & 4-(a+b)^2\end{pmatrix}

\left(\text{ Opérations: } (L1,L2) \rightarrow (L1,L1+L2)~\right)

Si |b|\neq|a|~:~rg(B)=4
Si a=-b\neq 0~:~rg(B)=2+rg\begin{pmatrix}  1 & 2\\0 & 4\end{pmatrix}=2+rg\begin{pmatrix}  1 & 1\\0 & 2\end{pmatrix}=2+rg\begin{pmatrix}  1 & 2\\0 & 2\end{pmatrix}=2+1=3

\bullet\text{ Deuxieme cas: }a=b
rg(B)=1+rg\begin{pmatrix} 1-a^2 & 0 & 1-a^2\\  0 & 0 & 0\\ 1-a^2 & 0 & 1-a^2\end{pmatrix}=1+rg\begin{pmatrix} 1-a^2 & 1-a^2\\  1-a^2& 1-a^2\end{pmatrix}

Si a=b=1 ou a=b=-1 : rg(B)=1
Si a=b\not{=}1 et a=b\not{=}-1 : rg(B)=2
\boxed{ rg(B)=\begin{cases} 1\text{ si } a =b=1\text{ ou } a=b=-1 \\ 2\text{ si } a =b\neq 1 \text{ et } a=b\neq-1\\ 3\text{ si } a=-b\neq 0\\ 4\text{ si } |a|\neq|b| \end{cases}}

3. rg(C)=rg\begin{pmatrix}1 & 1 & 1 \\b+c & c+a & a+b \\ bc & ca & ab \end{pmatrix}=rg\begin{pmatrix}1 & 0 & 0 \\b+c & a-b & a+c \\ bc & c(a-b) & b(a-c) \end{pmatrix}

\left(\text{ Opérations: } (C1,C2,C3) \rightarrow (C1,C2-C1,C3-C1)~\right)
On distingue trois cas :
Cas 1 : a,b et c sont deux à deux distincts

rg(C)=rg\begin{pmatrix}1 & 0 & 0 \\b+c & 1 & 1 \\ bc & c & b \end{pmatrix}=rg\begin{pmatrix}1 & 0 & 0 \\b+c & 1 & 0 \\ bc & c & b-c \end{pmatrix}=rg\begin{pmatrix}1 & 0 & 0 \\b+c & 1 & 0 \\ bc & c & 1 \end{pmatrix}=3
Cas 2 : si c=b\neq a ( ou a=c\neq b ou encore a=b\neq c )

rg(C)=rg\begin{pmatrix}1 & 0 & 0 \\b+c & 1 & 1 \\ bc & c & b \end{pmatrix}=rg\begin{pmatrix}1 & 0 & 0 \\b+c & 1 & 0\\ bc & c & 0 \end{pmatrix}=2
Cas 3 : si  a=b=c
Il est clair que dans ce cas: rg(C)=1
\boxed{ rg(C)=\begin{cases} 1\text{ si } a =b=c\\ 2\text{ si } c=b\neq a~,~a=c\neq b\text{ ou } a=b\neq c\\3\text{ si a, b et c sont sont deux à deux distincts}  \end{cases}}


exercice 10

Soit M\in\mathCAL{A}_3(\mathbb{K}).

Alors \det(M)=det(^tM)= \det(-M) = (-1)^3 \det(M)=-\det(M) donc 2\det(M) = 0 \Leftrightarrow \det(M) = 0

\boxed{\text{Donc M n'est pas inversible}}

(si la caractéristique de \mathbb K n'est pas 2 bien sur)

En considérant une base orthonormée de \mathbb{K}^n adaptée à Ker M, on peut écrire:
M=^{t}P M'P

Avec P\in\mathcal{O}_{3}(\mathbb{K}) et M' une matrice antisymétrique de la forme M'=\begin{pmatrix} 0 & 0 \\ 0 & M'' \end{pmatrix} telle que M^{''} une matrice antisymétrique inversible.
Puisque rg(M)=rg(M^{'})=rg(M^{''}), alors:
M \text{ est de rang paire }

D'autre part, M étant d'ordre 3, donc:
rg(M)\leq 3

On en déduit que:
\boxed{rg(M)=2\text{ ou }rg(M)=0}


exercice 11

Notons:

\begin{cases}\bullet\mathcal{B}_n~,~\mathcal{B}_p \text{ respectivement les bases canoniques de } \mathbb{K}^n \text{ et }\mathbb{K}^p\\ \bullet f\in\mathcal{L}(\mathbb{K}^n,\mathbb{K}^p) \text{ telle que } Mat_{\mathcal{B}_p,\mathcal{B}_n}(f)=A\\ \bullet   \begin{array}{clcl}  \tilde{f} : &\mathbb{K}^p  &\rightarrow &Im(f)\\&x&\mapsto&f(x)\\\end{array}     \text{ la restriction de f à Im(f) }\\ \bullet  \begin{array}{clcl}  i : &Im(f)  &\rightarrow &\mathbb{K}^n\\&y&\mapsto&y\\\end{array}  \text{ l'injection canonique } \end{cases}

On a ainsi:
f=i\circ\tilde{f}

Soit \mathcal{B} la base de Im(f), et soient B=Mat_{\mathcal{B},\mathcal{B}_n} (i) et  C =  Mat_{\mathcal{B}_p,\mathcal{B}} (\tilde{f}), on a alors:

\boxed{B\in\mathcal{M}_{n,r}(\mathbb{K})~,~C\in\mathcal{M}_{r,p}(\mathbb{K})\text{ et }A=BC}


exercice 12

Soit A=(a_{i,j})_{1\leq i,j\leq n} une matrice triangulaire supérieure et f l'endomorphisme de \mathbb{K}^n de matrice A dans la base canonique \mathcal{B} = (e_1, \cdots, e_n) de \mathbb{K}^n.
Soit \mathcal{B}^{'} =(e_n, \cdots, e_1), alors:
\mathcal{B}^{'} \text{ est aussi une base de } \mathbb{K}^n

Soient alors P la matrice de passage de\mathcal{B}^ à \mathcal{B}^' puis A^' la matrice de f dans la base \mathcal{B}^', il est clair que A^' est triangulaire inférieure.
Les formules de changement de bases permettent d'affirmer que:
A^{'} = P^{-1}AP

Donc:
\boxed{A \text{ et }A^{'} \text{ sont semblables }}
Publié le
ceci n'est qu'un extrait
Pour visualiser la totalité des cours vous devez vous inscrire / connecter (GRATUIT)
Inscription Gratuite se connecter
Merci à
monrow Posteur d'énigmes
/
monrow Posteur d'énigmes
pour avoir contribué à l'élaboration de cette fiche


Vous devez être membre accéder à ce service...

Pas encore inscrit ?

1 compte par personne, multi-compte interdit !

Ou identifiez-vous :


Rester sur la page

Inscription gratuite

Fiches en rapport

parmi 1675 fiches de maths

Désolé, votre version d'Internet Explorer est plus que périmée ! Merci de le mettre à jour ou de télécharger Firefox ou Google Chrome pour utiliser le site. Votre ordinateur vous remerciera !